CS计算机代考程序代写 \documentclass[11pt]{article}

\documentclass[11pt]{article}
\usepackage{amsmath}
\usepackage{amssymb}
\pagestyle{plain}
\usepackage{fullpage}
\usepackage{comment}
\includecomment{question}
\includecomment{solution}
\newcommand{\Implies}{\mbox{ IMPLIES }}
\newcommand{\Or}{\mbox{ OR }}
\newcommand{\AND}{\mbox{ AND }}
\newcommand{\Not}{\mbox{NOT}}
\newcommand{\Iff}{\mbox{ IFF }}
\newcommand{\True}{\mbox{T}}
\newcommand{\False}{\mbox{F}}
\def\reals{{\mathbb R}}
\def\ints{{\mathbb Z}}
\def\nats{{\mathbb N}}

\begin{document}
\begin{center}
{\bf \Large \bf CSC240 Winter 2021 Midterm Assessment Question 1}\\
YOUR NAME and STUDENT NUMBER
\end{center}

\medskip

\begin{enumerate}\item
\begin{question}
(8 marks)
Consider the predicate
$$(\forall x \in D.[R(x) \Iff \forall y \in D.A(x,y)]) \Implies \forall y \in D. (R(y) \Implies A(x,y)).$$
\end{question}

\begin{enumerate}
\item
\begin{question}
Give an interpretation with $D=\{1,2\}$ that makes this predicate true. Justify your answer.
\end{question}

\begin{solution}
{\bf Solution}:

\end{solution}

\item
\begin{question}
Give an interpretation with $D=\{1,2\}$ that makes this predicate false. Justify your answer.
\end{question}

\begin{solution}
{\bf Solution}:

\end{solution}
\end{enumerate}
\end{enumerate}
\end{document}